Sunteți pe pagina 1din 42

SEVERE INFECTION

Clinical problems

Module Authors Jordi Rello Michel Wolff Peter Wilson


Department of Critical Intensive Care Unit, Department of
Care, University Hospital Bichat-Claude Bernard Microbiology,
Joan XXIII, Tarragona, Hospital, Paris, France University College
Spain London, London, UK

Eva-Maria Weiss- Roland Dietler*


Guillet* Dept of Intensive Care
Spitäler FMI AG, Spital Medicine, University
Interlaken, Unterseen, Hospital Bern,
Switzerland Switzerland

*For the Bernese PACT Roland Dietler, Raymond Friolet, Roger Lussmann, Hans-Ulrich Rothen,
Working Group (in Kay Stricker, Jukka Takala, Eva-Maria Weiss-Guillet, Jeannie Wurz
alphabetical order)

With input/comments Jorge Sole Violan, Las Palmas, Spain (Section on Infection)
from Metka Derganc, Ljubljana, Slovenia (ESPNIC)
Hans Kieft, Zwolle, the Netherlands
and the following members of the PACT Steering Committee: Dermot
Phelan, Graham Ramsay, and Arend Woittiez

Editor-in-Chief Graham RAMSAY


Chairman of the PACT Steering Committee
Medical Director, West Hertfordshire Hospitals
NHS Trust, UK
Past President, European Society of Intensive
Care Medicine

Module Editor Graham Ramsay, West Hertfordshire Hospitals


NHS Trust, UK and AB Johan Groeneveld, Free
University Hospital, Amsterdam, the Netherlands

Educational Editor Lia Fluit, Medical Education Training and


Consultancy (METC), Nijmegen, the Netherlands

Editorial Manager Kathleen A Brown, Triwords Limited, Tayport, UK

Editorial Secretary Laurence Van den Bossche, ESICM, Brussels,


Belgium

Severe infection

Clinical problems
LEARNING OBJECTIVES
After studying this module on Severe infection, you should be able to:

1. Recognise the patient with severe community-acquired and nosocomial infection


2. Develop a therapeutic strategy of source control and antibiotic use
3. Choose appropriate initial antibiotic therapy
4. Reassess initial empiric management and make appropriate changes
FACULTY DISCLOSURES
The authors of this module have not reported any disclosures.

DURATION
9 hours

Copyright©2007. European Society of Intensive Care Medicine. All rights reserved.


ISBN 978-92-95051-90-4 - Legal deposit D/2005/10.772/37

INTRODUCTION
A substantial fraction of patients are admitted to the ICU because of severe community-acquired
pneumonia, meningitis, urosepsis, or infections requiring a surgical approach. Infections occurring >48
hours after hospital admission are by definition nosocomially-acquired. They are documented with an
overall incidence of about 10%. Twenty-one per cent of patients in the EPIC Study (see reference
below) had an infection. Nosocomial pneumonias contributed with 46.9%, followed by urinary tract
infections with 17.6%, and bloodstream infections with 12%.

Nosocomial infections in neonatal and paediatric ICUs have different epidemiology. In infants, blood
stream infections (BSI) predominate (34% of infections), followed by nosocomial pneumonias (18%)
and urinary tract infections (12%). With increasing age, the BSI rate decreases to 21% at ages 5-12
years, while the rate of pneumonias increases to 26% and that of urinary tract infection to 17%.
This prevalence demonstrates a strong correlation with ICU length of stay, mechanical ventilation,
trauma, central venous, pulmonary artery, and urinary catheterisation, and stress ulcer prophylaxis.
Nosocomial infections increase the risk of ICU death. This module will focus on the diagnosis and
management of severe infection. You will find general reviews about infection in intensive care in the
following references.

1/ EXAMINING THE BASIS FOR SUSPECTING A SEVERE INFECTION

How to recognise the patient with severe infection


Any process that results in the release of the pro-inflammatory cytokines (e.g. tumour necrosis factor-α
(TNF-α), interleukin 1, interleukin 6) will cause a non-specific inflammatory response, which may
include fever, leukocytosis and increased levels of C-reactive protein. This response has been referred
to as the 'systemic inflammatory response syndrome', or SIRS. It may be induced by infection but also
trauma, burns, surgery, myocardial infarction, pulmonary embolism, acute pancreatitis, haematoma,
gastrointestinal haemorrhage, drug fever, alcohol and drug withdrawal syndromes, and reactions to
transfusions of blood products.
Patients with sepsis, defined as a systemic inflammatory response to infection, may present in
approximately 35% of cases with normal body temperature or even, in up to 10%, with hypothermia.
Sepsis, severe sepsis, and septic shock represent increasingly severe stages of the same
pathophysiological process with a progressive increase in system-wide organ dysfunction and failure,
culminating in death.
From the clinical point of view, detecting infection as the cause of an inflammatory response is
therefore crucial: the presence of infection increases morbidity and mortality, and an infection can be
specifically treated. Early treatment of infection by removing a focus and appropriate antibiotic
treatment will improve the outcome. For more information see the PACT modules on
Immunocompromised patients and Sepsis and MODS
Clinicians face major diagnostic challenges in their approach to a febrile patient. First, the non-
infectious causes of inflammation must be differentiated from the infectious ones. Second, in patients
with suspected infections, community-acquired infections should be differentiated from
nosocomiallyacquired diseases. Third, colonisation must be differentiated from infection in a febrile
patient with multi-organ dysfunction. And fourth, the appropriateness of host defence should be
considered, especially in patients who are neutropenic or immunocompromised.
A careful history and physical examination usually give clues to the nature of the severe illness
requiring admission to the ICU. Infectious diseases often have a fulminant acute onset and the patient's
clinical presentation deteriorates rapidly. Immediate performance of relevant investigations and a
focused diagnostic work up help identify potential infection and guide the timely initiation of appropriate
antibiotic therapy.

Confirming infection
Local symptoms and extent may give a direct indication for suspecting infection. More commonly, the
intensivist is confronted with a patient with an acute or impending organ dysfunction, e.g. dyspnoea,
acute circulatory failure, decreased level of consciousness, or an acutely deteriorated general
condition. In this case, diagnosis of infection as a precipitating factor should accompany the support of
organ functions and organ-specific diagnostics.
On the other hand, signs of new infections in intensive care patients can be very subtle, ranging from
new or worsening organ dysfunctions or fever to acute circulatory failure. Again, a thorough clinical
evaluation and the selection of appropriate diagnostic techniques are important elements in the
management of these patients.

Diagnostic approach

PRACTICE Suspect, examine, find the focus by clinical examination and imaging; drain,
sample, and treat. Microbiological samples are essential. They help to confirm the
diagnosis, decide on treatment, determine antibiotic susceptibilities, and guide
future empirical therapy.

The clinician's initial assessment of the febrile patient should always be based on history
and complaints, clinical appearance, and a thorough physical examination focusing on
organ function changes (e.g. neurological deterioration, worsening gas exchange and/or
changes in pulmonary secreta, ileus) and local findings (e.g. wounds, catheters,
inflammation of soft tissues, thrombophlebitis).

Investigations

It is usually possible to target the investigations based on the clinical history and physical examination.

Complete blood cell count including platelet counts, arterial blood gases, lactic acid level, serum
electrolytes, cardiac enzymes, blood urea nitrogen and creatinine, liver function tests, coagulation
parameters.
Two sets of blood cultures by separate needle sticks from different venipuncture sites; cultures from
intravenous lines in suspected catheter-associated sepsis; three to five blood cultures are required if
endocarditis is suspected. In patients with central venous catheters, a quick method for identifying the
micro-organisms in blood by staining the buffy coat by acridine orange or Gram stain has been shown to
be useful (see references on the next screen).

Urine analysis and culture in suspected urosepsis


Analysis, Gram stain and culture of the cerebrospinal fluid in suspected meningitis, encephalitis
Sputum, tracheal aspirates, protected specimen brush (PSB), or bronchoalveolar lavage (BAL) cultures
and Gram stain in suspected pneumonia
Cultures and Gram stain of relevant body exudates:
Loculated fluids (pleural, peritoneal liquid)
Abscesses
Wound material
Transcutaneous drains
Stool cultures in suspected colitis
Electrocardiogram
Chest roentgenogram
Transoesophageal echocardiogram, abdominal roentgenograms, computed tomography of head, sinuses,
chest, or abdomen/pelvis, as clinically required

Every ICU should develop a diagnostic protocol, taking into account the prevalence of conditions and the
diagnostic yield of individual tests. Although most sources of infection will be detected with a systematic
approach, some patients will have negative results for all tests. If the patient has clinical evidence of severe
infection with systemic involvement, characterised by hypotension, metabolic acidosis, or worsening organ
function, initiation of broad-spectrum antibiotic therapy should not be delayed.

Use the diagnostic protocol in your unit for the next five patients that you see.
Compare the discharge diagnosis with what you recorded. Check the antibiotics
used against the protocol recommendations.

Blood culture

Blood culture should be performed for patients with new fever, even when the clinical signs do not strongly
suggest an infectious cause. The procedure is to clean the site of venipuncture with either 10% povidone-
iodine or alcohol. Some American studies have found 2% chlorhexidine more effective than povidone-iodine
and alcohol.

Draw at least 10-15 ml blood in adults; change needles if injecting into a blood culture bottle. In children,
smaller amounts should be taken. Two samples in the initial 24 hours are usually sufficient (endocarditis:
three to five samples). Each sample should be withdrawn by separate venipuncture.

NOTE Bacteria are rapidly cleared from the blood. Development of fever usually follows
an episode of bacteraemia by 30-90 minutes, so blood cultures should be taken as
soon as possible following a spike of fever.

Involvement of particular sites

Community-acquired infections

The severity of illness will be determined by underlying systemic disease or impaired host defences. In
immunosuppressive conditions (HIV patients, neutropenic patients after chemotherapy, transplant patients,
and patients with auto-immune disorders receiving steroids, methotrexate or other immunosuppressive
agents) the clinician has to consider that the spectrum of pathogens involved may be changing, that
normally opportunistic germs may cause overwhelming infections, and that device-related infections may be
involved. For more information see the PACT module on Immunocompromised patients
The most frequently diagnosed community-acquired infections leading to ICU admission include severe
pneumonia, meningitis, and sepsis with intra-abdominal sources. Necrotising fasciitis is sporadic and
uncommon but extremely challenging for the clinician. The infections are organised by anatomical site on
the subsequent screens.

PRACTICE Intracerebral infections

Meningitis
Meningitis is the most common form of intracranial infection. Acute bacterial meningitis remains a significant
cause of morbidity and mortality. Classical signs of meningitis are fever, headache, photophobia, and
nuchal rigidity. Focal neurological deficits, central nerve palsies of the cranial nerves IV, VI and VII may be
present. Generalised or focal seizures are frequent complications. A diabetes insipidus or a syndrome of
inappropriate ADH secretion (SIADH) can be a temporary problem.
There may be cutaneous manifestations, such as petechial or purpuric rash as an expression of
disseminated intravascular coagulopathy (typical in meningococcaemia, rare in Streptococcus pneumoniae
and Haemophilus influenzae meningitis, possibly seen in meningitis due to Staphylococcus aureus),
resulting in bleeding disorders, shock, extravasation, and multi-organ dysfunction. At the extremes of age,
however, the typical clinical presentation can be lacking or absent, and changes in mental status
(confusion, delirium, declining consciousness) associated with fever may be the leading symptoms.

NOTE Any patient with changes in mental status and fever should be assumed to have
meningitis and treated as such until the diagnosis is excluded.

Pathophysiology

Infection occurs with haematogenous spread and crossing of the blood–brain barrier after colonisation and
invasion, usually of the nasopharyngeal mucosa. An otitis media or a sinusitis may be the original local
infection focus, pneumonia or endocarditis a distant one. Transfer of germs is also possible via a
cerebrospinal fluid (CSF) cranial fistula or a ventriculoperitoneal shunt. Once in the subarachnoid space,
bacterial multiplication may proceed rapidly, as the CSF is a poor immune environment.
In the CSF, a number of bacterial components (lipopolysaccharide and others) initiate an inflammatory
change with release of proinflammatory factors (TNF-α, IL-1β, IL-6, NO), and a vasogenic and cytotoxic
oedema develops. Obliteration of the subarachnoid space by pus and the obstruction of the arachnoid villi
by white cells can result in acute hydrocephalus. Subdural and epidural empyema and brain abscess as
further complications require surgical drainage.

Infecting organisms

The causative organism will vary according to the population, age, and underlying condition.
Neisseria meningitidis and Streptococcus pneumoniae are the most common pathogens. Haemophilus
influenzae meningitis has declined due to vaccination in young children, but may be responsible for
meningitis in patients with immunosuppressive illnesses or therapies. In the elderly and
immunocompromised, Listeria monocytogenes, an aerobic Gram-positive bacterium, is also commonly
implicated. Tuberculous meningitis is most commonly a complication of post-primary infection in individuals
with advanced age, malignancy, and HIV. Alcoholism and malnutrition are further risk factors.
Meningitis in the very young may be due to Group B Streptococcus and P. aeruginosa is more common
Gram-negative bacilli (E.coli, Pseudomonas sp.), and Listeria in immunodeficient infants and
monocytogenes. in nosocomial infection than in
community-acquired infection
In patients with a ventriculoperitoneal shunt or a persistent CSF cranial
fistula after trauma, Staphylococcus aureus, coagulase-negative
staphylococci, enterococci and Gram-negative bacilli are common
causes.

Diagnosis and management

Deterioration may be rapid, so prompt treatment is essential, even before a diagnosis is confirmed or
results of CSF analysis or CT scan are obtained.
In bacterial meningitis the CSF analysis reveals a cloudy fluid, consisting of an increased white blood cell
count with a predominance of polymorphonuclear leukocytes, a high protein level, and a low glucose
concentration in relation to serum value (<50%). In tuberculous meningitis, however, a lymphocytic
pleocytosis is typical.

NOTE Empirical antimicrobial therapy should be initiated within 20 minutes of


encountering the patient with acute meningitis.
What are the risk factors for bacterial meningitis in children?

Should empiric antibiotic treatment be withheld until microbiological samples are taken to prevent
sterilisation of samples?

Algorithm for the


management of patients with
the acute meningitis
syndrome

In the treatment of individuals with severe meningococcal sepsis, protein C replacement therapy should be
considered. In its activated form, protein C is not only an anticoagulant but also a relevant modulator of the
inflammatory response. In fulminant meningococcaemia, as in conditions of severe sepsis, levels of protein
C are markedly reduced. Intravenous administration significantly improves the rate of survival and clinical
outcome, but with a slight increase in the risk of bleeding.
In children, only one study has been published on the safety of drotrecogin alfa in severe sepsis, showing
no increase in the risk of bleeding. Several clinical studies are under way, but for the time being, the routine
use of drotrecogin alfa is not yet recommended. See Barton reference below.

Encephalitis

The acute encephalitis syndrome features a triad of fever, headache, and neurological dysfunction (altered
consciousness, mental aberrations, focal findings, focal or generalised seizures, involvement of the
hypothalamic-pituitary axis). Inflammation of neural parenchyma is the pathological feature, and if the
coverings of the brain are included, the term meningoencephalitis is used.

Diagnosis and management

It is important to begin diagnostic procedures early. In imaging, MRI is superior to computed tomography.
The CSF profile consists, in general, of a lymphocytic pleocytosis, a slightly elevated protein level, and a
normal glucose level. Cultures and PCR amplification of viral nucleic acid from the CSF fluid should be
performed. The distinction of herpes simplex (HSV) from other viruses that cause encephalitis is particularly
important, because specific antiviral therapy is available. Acyclovir is a relatively safe drug, thus immediate
empirical administration in cases of suspected herpes simplex encephalitis is justified. Other non-viral
causes of encephalitis have to be considered (see table)
When encephalitis is accompanied by cerebral swelling and raised ICP, corticosteroids are a reasonable
intervention. Seizure activity frequently complicates the clinical course. It can lead to permanent damage of
the vulnerable neurons, and can exacerbate problems with elevated ICP. Prompt administration of
phenytoin is recommended, maybe in combination with benzodiazepines.

NOTE An antiviral therapy started early can dramatically reduce the mortality and
morbidity associated with herpes simplex encephalitis.

T HINK More and more patients are immunocompromised because of HIV, vasculitis treatment,
or organ transplantations. How can this influence the spectrum of pathogens for
meningitis and encephalitis?

Acute disseminated encephalomyelitis (ADEM) is a syndrome characterised by focal neurological signs, a


rapidly progressive course, and a prodromal history of febrile illness or vaccination. The absence of fever at
the onset of symptoms helps to distinguish ADEM from infectious encephalitis.

Outcome

The incidence and severity of sequelae in acute meningitis and encephalitis depend strongly on the
pathogen and the delay of therapy, and are related to the age of the patient and the level of consciousness
at initiation of therapy.

Severe upper respiratory infections

Acute laryngitis and laryngotracheobronchitis (Croup)

Influenza virus, rhinovirus, and adenovirus are the most frequent causes of acute laryngitis. Secondary
bacterial invasion with Moraxella catarrhalis and Streptococcus pyogenes is possible. Obstruction of the
airway, however, is rare in adults and must be distinguished from acute bacterial epiglottitis.
Croup, or acute laryngotracheobronchitis, is an age-specific viral infection with a peak occurrence in the
second year of life, and causing a severe inflammation in the subglottic area. Oxygen supply,
corticosteroids (preferably systemic) and nebulised racemic or l-adrenaline are the mainstays of treatment
(reference below). Close clinical observation is mandatory for early recognition of the necessity of a
mechanical airway.

Epiglottitis

A sudden onset of hoarseness, dysphagia, drooling, and an inspiratory stridor are the leading symptoms in
acute epiglottitis, and the direct visualisation typically shows an oedematous 'cherry-red' epiglottis.
Haemophilus influenzae is the most frequently responsible pathogen; other possible implicated agents are
pneumococci, staphylococci, and streptococci. The course of acute epiglottitis may be fulminant, with
possible abrupt, complete airway obstruction, so the maintenance of an adequate airway should be the
primary concern. Cultures from the blood and the epiglottis should follow, and the patient should receive an
initial antibiotic treatment either with cefotaxime, ceftriaxone or ampicillin. The role of epinephrine and
steroids is not established in acute epiglottitis.

Severe head and neck infections

Severe infections of the head and neck are consequences of uncontrolled odontogenic, oropharyngeal, or
otolaryngeal infections. They may involve adjacent structures around the face, extending to spaces of the
suprahyoid and infrahyoid region, and to the mediastinum, always following planes of least resistance.
Dissemination by haematogenous spread, cavernous sinus thrombosis, suppurative jugular
thrombophlebitis (postanginal sepsis – Lemierre syndrome) and internal carotid artery erosion, maxillary
sinusitis, and osteomyelitis may occur.
The clinical presentation depends on which compartment is involved. Swelling, painful chewing and
swallowing, trismus, impaired respiration due to obstruction of the tongue, or laryngeal oedema with
manifestations of sepsis may be apparent.

Diagnosis and management

In the diagnostic approach, computed tomography scanning (CT) is particularly useful for the localisation
and extension of infection. For accurate microbiologic diagnosis, Gram-stained smears, aerobic and
anaerobic cultures of pus (in close-spaced infections needle aspiration by an extra-oral approach is
preferable in order to avoid contamination by the normal resident flora) or tissue biopsy specimens, and
blood should be obtained. The bacteriology of odontogenic and pharyngeal infections is largely
polymicrobic, with oral anaerobes and Gram-negative bacilli.
The principles of management of deep fascial space infections include administration of appropriate
antibiotics and prompt surgical drainage if suppuration is noted.

ANECDOTE A young woman presented with painful swallowing and neck movements, an inspiratory
stridor and dyspnoea, and manifestations of sepsis with a rapid clinical deterioration two
weeks after tonsillitis. Physical examination revealed a brawny, indurated swelling, and a
tender submandibular adenopathy on the left side. After maintenance of the airway by
fibre optic intubation, a CT scan of the neck revealed a deep tissue swelling with a
parapharyngeal collection, and a thrombus formation of the internal jugular vein. The CT
scan of the chest demonstrated lung nodules on both sides, consistent with septic
emboli. The immediate surgical approach included generous exploration and drainage,
and a tracheostomy under broad antibiotic covering with ceftriaxone and metronidazole.
A resection of the internal jugular vein was not performed. The blood culture revealed
Fusobacterium necrophorum. Clinical improvement with a favourable outcome was seen
after weeks of intensive care treatment, complicated by an ARDS, a septic myocardial
failure and renal failure. Because of an associated critical illness polyneuropathy the
transfer to a rehabilitation unit was organised. At discharge the thrombus of the internal
jugular vein had resolved.

Disorders of the mediastinum

Mediastinitis

Acute mediastinitis is most frequently due to oesophageal perforation, either spontaneously, as a


complication of endoscopic interventions and the insertion of a Sengstaken-Blakemore tube, or by trauma.
Patients present with severe chest pain and dyspnoea, and are acutely and severely ill, as the initial
chemical necrotising injury is followed by bacterial infection.
Odontogenic infections involving the sublingual and submandibular spaces and infections involving the
lateral and retropharyngeal space may also spread to the mediastinum, and mediastinitis may be
secondary to cardiothoracic surgery.
Treatment requires immediate surgical exploration of the mediastinum, oesophageal repair in case of
perforation, debridement, and drainage of the mediastinum and pleural space. Empirical antibiotic regimens
are based on the underlying aetiology and should cover the major pathogens. Oropharyngeal aerobic and
anaerobic bacteria have to be considered in mediastinitis secondary to oesophageal perforation or
infections of the head and neck, and staphylococci and Gram-negative enteric bacilli after cardiothoracic
surgery until definitive culture results are available.

Severe lower respiratory infections

Pneumonia

Clinical signs and infecting organisms


Severe community-acquired pneumonia may present with a typical or an atypical syndrome. Sometimes,
however, the two syndromes may be less distinct.

In immunosuppressive conditions the risk of pneumonia depends not only on the type of
immunosuppression but also on its extent and duration. In neutropenia, Gram-negative bacilli are
uncommon above a neutrophil count of 500 cells/mm3, but increase in frequency, especially Pseudomonas
aeruginosa, if the neutrophil count falls below 500 cells. Patients with neutropenia of long duration are at
risk for invasive aspergillosis.

NOTE Patients with aplasia may not develop pulmonary infiltrates with pneumonia.

In patients with HIV and a CD4 count under 200 cells/mm3, Pneumocystis jiroveci pneumonia may develop.
T lymphocyte deficiencies also predispose for viral pneumonias.
Tuberculosis, pulmonary infections with nocardia, and dimorphic fungi (Histoplasma, Coccidioides,
Blastomyces) have to be considered.

Diagnosis

PRACTICE Diagnosis of severe community-acquired pneumonia (CAP) should follow the usual
steps in the immunocompetent as well as in the immunocompromised host: history,
physical examination, non-invasive tests including radiological assessment, and invasive
procedures.
Despite intensive investigation, the causative agent is not isolated in as many as 40% of
cases of community-acquired pneumonia. This does not mean, however, that
identification should not be attempted. Knowledge of the aetiology is useful in guiding
patient management and for epidemiological purposes. In cases of suspected lower
respiratory tract infection, secretions should be obtained for examination and culture.

Obtaining lower respiratory secretions

Expectoration
Hypertonic saline induction
Deep tracheal suctioning
Bronchoscopic and non-bronchoscopic protected brush
Bronchoscopic or non-bronchoscopic alveolar lavage
Transthoracic needle aspiration (TNA)

Blood cultures are positive in about 10-30% of patients with serious CAP. The isolation of a respiratory
pathogen from a blood culture is highly specific for diagnosing bacterial pneumonia.
Pulmonary secretions should be processed within two hours so that any fastidious organisms, such as
Streptococcus pneumoniae, do not die. The presence of elastin fibres in tracheal aspirate has been found
to improve specificity to 73%, but they may be absent in pneumococcal pneumonia or pneumonia caused
by Mycoplasma pneumoniae. Early microscopic identification of organisms within the alveolar cells (≥2%
intracellular organisms) recovered by bronchoalveolar lavage (BAL) can be helpful in establishing the
presence of pneumonia. Fibre optic bronchoscopy can be used to collect brush or lavage specimens from
the lower airways, which are free of contamination from pharyngeal flora. A colony count of 104 CFU/ml in
the withdrawn fluid from BAL is diagnostic, but patients with bronchitis can reach these counts without
pneumonia. A Gram stain of the fluid is valuable in showing the bacteria, elastin fibres, and neutrophils that
are more common in pneumonia.
The diagnosis of Legionella pneumophila can be made by detection of antigen in urine.
Other tests should be carried out in appropriate situations:

Potassium hydroxide wet mount for detecting fungi


Enzyme immunoassay (EIA) for influenza A virus
Acid-fast stain for mycobacteria
Fluorescent stain for Pneumocystis jiroveci

Therapy

In normal hosts presenting with severe community-acquired pneumonia who are hypotensive and in shock,
the infecting organism is most likely S. pneumoniae; in alcoholics it is Klebsiella pneumoniae. Legionnaire's
disease may also be severe in its clinical presentation. As delay in initiation of appropriate antibiotic therapy
is associated with a poor outcome, a reasonable empiric therapy therefore includes β-lactam antibiotic,
usually amoxicillin-clavulanate or a third-generation cephalosporin, and a macrolide (clarithromycin).
Levofloxacin can be given alternatively.
In the compromised host, antipseudomonal coverage is necessary. Therefore, initial therapy, preferably,
should be begun with a fourth-generation cephalosporin or meropenem, a carbapenem. In severe
pneumonia with diffuse or interstitial infiltrates, trimethoprim-sulfamethoxazole should be included if no
Pneumocystis jiroveci prophylaxis has being given. Ganciclovir may be used empirically in transplant
recipients at high risk.

Aspiration

Aspiration of vomited gastric contents causes primarily a chemical injury to the lung, with loss of the
alveolocapillary integrity and exudation of fluid and protein, causing pulmonary oedema and significant
intrapulmonary shunting. The lung, however, will be more vulnerable to secondary infections.
In the treatment of aspiration pneumonia, oral anaerobic bacilli, the most common responsible infective
pathogens, are covered with amoxicillin-clavulanate.

Complications related to pneumonia

Pleural empyema

In adults, the most common causes are Staphylococcus aureus, Streptococcus pneumoniae and
Streptococcus pyogenes. Immunocompromised patients have a higher risk of pleural infections caused by
fungi and aerobic Gram-negative bacilli. Persistence of fever and leukocytosis despite adequate
antimicrobial therapy, and the accumulation of pleural fluid, should suggest the diagnosis.
Analysis and cultures of the pleural fluid are mandatory, and percutaneous drainage is the treatment of
choice. Intrapleural thrombolytic therapy, thoracoscopy, decortication or an open drainage procedure may
be necessary.

Lung abscess

Failure to respond to adequate antibiotic therapy may be due to the formation of a lung abscess. Most lung
abscesses develop following aspiration pneumonia or septic emboli. Mixed aerobes and anaerobic bacteria
are responsible in most of the cases, but various fungal infections, Legionella, nocardiosis and
Pneumocystis jiroveci may lead to abscess formation too.
In primary lung abscess, tuberculosis and a necrotising carcinoma have to be excluded. Diagnostic
bronchoscopy or percutaneous needle aspiration are the diagnostic procedures of choice, and
percutaneous drainage is the indicated treatment when failing to respond to medical measures.

Endocarditis
Patients at risk

Predisposing conditions for infective endocarditis are underlying congenital heart diseases, with the
bicuspid aortic valve as the most common one, mitral valve prolapse, rheumatic heart disease, and
degenerative cardiac lesions. The mitral and aortic valves are predominantly affected. The valves on the
right side of the heart are rarely involved in non-intravenous drug abusers. Prosthetic valve endocarditis
develops in 2-3% of patients having valve replacement.

Clinical signs and complications

General signs of infection and altered murmurs in cardiac auscultation are leading symptoms. Septic emboli
may travel to the meninges and brain, the coronary arteries, the spleen, kidney, the skin (Osler's spots,
Janeway lesions), and less commonly to the bones and joints. Cerebral emboli occur in 30% of cases of
acute infective endocarditis and lead to abscesses and multiple infarctions with possible secondary
haemorrhage. Mycotic aneurysms, which are life-threatening in 2.5% of all cases, arise from direct bacterial
invasion of the blood vessel wall. The drug abuser with right heart endocarditis may present with
pneumonia and lung abscesses.
15% to 65% of all patients may develop congestive heart failure as a result of valvular damage. Aneurysm
of the sinus Valsalva and intraventricular abscesses may be responsible for abnormalities in electric
conduction.

NOTE Transient bacteraemia during a dental procedure may cause infective endocarditis
in patients at risk, with symptoms beginning 14 days after the intervention.

Pathophysiology

Endothelial damage leads to blood turbulence, followed by platelet and fibrin deposition. In transient
bacteraemia (or fungaemia) microbial attachment to the thrombus follows, forming a vegetation. Successive
layers of fibrin, platelets, and micro-organisms enlarge the vegetation. Microbial colonies within the
vegetation are relatively resistant to host defences and antibiotic penetration, so often require prolonged
high-dose antimicrobial therapy.

Infecting organisms

The most common organism producing acute endocarditis is Staphylococcus aureus, in both normal hosts
and intravenous drug abusers. Coagulase-negative staphylococci are the most frequent pathogen in
prosthetic valves as well as in other types of implanted materials. Streptococcus viridans is responsible for
about 70% of subacute cases, followed by Group D streptococci. The growth of S. bovis is strongly
associated with inflammatory bowel disease and colon cancer. Rarely, S.viridans and Group D streptococci
produce acute disease. Gram-negative organisms have a decreased ability to adhere to valvular
endothelium and therefore rarely produce valvular infections.
Polymicrobial and fungal endocarditis have become more prevalent in cardiac surgery patients and
intravenous drug abusers in the last decade.

Diagnosis

PRACTICE The optimum number of blood cultures should be no less than three and no more than
five. As persistent bacteraemia >30 min is the hallmark of endovascular infection,
cultures can be obtained over a period of thirty minutes (12 hours in the subacute
forms), but have to be taken from different insertion sites under sterile conditions.
Transoesophageal echocardiogram is the method of choice to visualise vegetations.
However, 10% to 20% of infected valves have no vegetations by echocardiography.
Explain the link between dental surgery and endocarditis.

Therapy

The initial antibiotic therapy, based on clinical and physical examination, requires high dosages, and should
be a combination of bactericidal antibiotics with synergistic activity. A combination of amoxicillin-clavulanate
and gentamicin is recommended in acute endocarditis; penicillin and gentamicin in subacute forms. In
patients with known allergic reactions to penicillin, a combination of vancomycin and gentamicin is the
alternative. Prosthetic valves require a combination of vancomycin, rifampicin, and gentamicin. The initial
choice of antibiotics should be adjusted when definite microbiological data are available.
In patients with persisting fever despite appropriate antibiotic treatment, a vigorous work up for mycotic
aneurysms and other extracardiac infections should be conducted. Fungal endocarditis must be considered
when a patient presents with signs and symptoms of infective endocarditis but has negative blood cultures
and fails to respond to standard antibiotic therapy.
Indications for surgical intervention are progressive valvular destruction, persistent sepsis despite adequate
antibiotic therapy, paravalvular and myocardial abscesses, and rupture of aneurysm or sinus of Valsalva.

NOTE Antibiotic prophylaxis is necessary in patients at risk for endocarditis who


undergo procedures associated with transient bacteraemia.

Myocarditis

Patients with myocarditis may suffer from a severe reduction in ventricular function as a result of myocardial
inflammation, and complain of symptoms related to heart failure or symptomatic dysrhythmias. ST-segment
and T wave changes in correlation with myocardial enzyme release generally appear along with the signs of
cardiac dysfunction.
Infectious myocarditis is considered in the setting of a febrile illness or after an upper respiratory tract
infection, with viruses as the major agents implicated. Bacterial pathogens that invade the bloodstream may
cause metastatic foci in the myocardium or result in toxin-related myocarditis. In immunocompromised
patients, systemic aspergillosis, candidiasis, cryptococcosis, and disseminated toxoplasmosis may involve
the heart.
Echocardiography is useful in detecting and quantifying wall motion abnormalities and ventricular dilatation,
but the 'gold standard' for diagnosis is endomyocardial biopsy.
Treatment should be directed at the specific aetiologic agent whenever possible, and the symptoms of heart
failure. In general, viral myocarditis is a self-limiting disease that resolves. Some patients, however, don't
recover because of irreversible myocardial injury. In those patients who have ejection fractions less than
25% and who fail to respond to maximal medical therapy, mechanical circulatory support devices and
cardiac heart transplantation provide the only effective therapy for long-term survival.

T HINK of non-infectious causes of dilated cardiomyopathy.

For more information, see the PACT module on Heart failure

Pericarditis

Non-infectious processes and a wide variety of infections may result in pericarditis. Most viruses infecting
the heart affect both the myocardium and the pericardium. Purulent pericarditis is frequently associated with
pleuropulmonary infections. In immunocompromised patients tuberculous and fungal pericarditis may be
seen.
In the clinical presentation, chest pain, typically aggravated by breathing and lying supine, is an important
feature, especially in viral pericarditis. Bacterial pericarditis is accompanied with the signs of a severe
systemic infection. Pericardial friction rub is the classic physical finding, and ECG changes reflect the
subepicardial inflammation. Pericardial effusions may lead to haemodynamic compromise, and
pericardiocentesis is needed. In suspected purulent pericarditis, surgical drainage and appropriate antibiotic
therapy are the essential mainstays of treatment.
Non-steroidal anti-inflammatory agents are often successful in relieving the pain.

T HINK of non-infectious causes of pericarditis

Severe intra-abdominal infections

For further information, see the PACT module on Abdominal problems

Neutropenic enterocolitis

Clinical signs

Neutropenic enterocolitis or typhlitis is a potentially life-threatening, necrotising inflammation of


predominantly the cecum and the right colon, involving surrounding tissues. The condition is associated
with severe neutropenia (absolute neutrophil count <500/mm3). Bowel wall inflammation and localised
necrosis subsequently result in secondary infections, often polymicrobial in nature, due to translocation with
enteric and/or opportunistic organisms.
The clinical presentation is non-specific, with pyrexia, abdominal pain, vomiting, bloody diarrhoea, a right
iliac fossa or generalised tenderness, reduced bowel sounds, and in severe cases with septic shock,
multiple organ dysfunction and multiple organ failure.

Diagnosis

CT imaging is the diagnostic method of choice. It is more accurate than ultrasound in determining bowel
wall thickening and the extent of the colitis, and in showing evidence of pericecal fluid, soft tissue
inflammatory changes, pneumatosis coli, or free intraperitoneal air.

Management

A conservative approach toward management is advised, and includes gastric decompression, parenteral
nutrition, broad-spectrum antibiotics, balanced fluid and blood product replacement, and Granulocyte
Colony Stimulating Factor (GCSF)-therapy. With the return of neutrophil counts healing may be expected.
The only indications for surgery include free intraperitoneal perforation, continued bleeding despite
correction of any coagulopathy, and uncontrolled septic shock despite aggressive medical treatment.

What are the most common pathogens found in intra-abdominal infections?

Urosepsis

Community-acquired urosepsis occurs only in compromised patients (cystitis in non-leukopenic patients


with diabetes mellitus, vasculitis, steroid therapy, etc.) or in hosts with pre-existing renal disease or
structural abnormalities of the urinary tract.

Clinical signs and infecting organisms

Besides clinical symptoms of sepsis, the history and physical examination reveal flank pain and marked
tenderness on deep pressure in the costovertebral angle or on deep abdominal palpation.
The infections are usually caused by enteric Gram-negative bacilli.

Diagnosis and management

The diagnosis is confirmed by pyuria with leukocyte casts in the urine, bacteria in Gram-stained urine, and
isolation of the infecting organism from blood or urine culture.
Empirical antibiotic therapy with amoxicillin-clavulanate, or alternatively with ceftriaxone or ciprofloxacin, is
essential. In cases of septic shock, the addition of an aminoglycoside is recommended. Treatment also
requires assessment and removal of urinary tract obstruction.

Skin and soft tissue infections

Patients at risk

Severe community-acquired skin and soft tissue infections (SSTIs) occur most often in compromised hosts
with diabetes, peripheral vascular disease, corticosteroid therapy, advanced cancer, or AIDS, or in
alcoholics. But healthy individuals may also develop SSTIs after minor trauma (lacerations, puncture
wounds, insect bites), skin lesions (furuncle), stab wounds, burns, or surgery. Bedridden patients may
develop SSTI as a complication of decubitus ulcers.

General clinical signs

The urgency and extent of treatment should be determined by the following parameters:

The presence of systemic symptoms


Macroscopic appearance
Depth of microbial invasion (assessed through physical examination and/or surgical exploration): skin,
fatty subcutaneous tissue, muscular fascia and muscle
Time from onset to overt disease

Many SSTIs progress rapidly and may cause serious systemic complications if therapy is not instituted
promptly. The table below gives signs of SSTIs.

Erysipela and cellulitis

Clinical signs

Erysipelas is an unmistakable type of superficial cellulitis of the skin with a painful, erythematous,
oedematous, bright red appearance and prominent lymphatic involvement. Bacteraemia occurs in about 5%
of patients.
Cellulitis is a spreading infection of the skin extending deeper to subcutaneous tissues than erysipelas.
Early clinical differentiation of cellulitis from necrotising fasciitis may be difficult. The presence of marked
systemic toxicity out of proportion to the local findings and markedly elevated serum creatinine
phosphokinase should alert the clinician.
Progressive bacterial synergistic gangrene is a special form of slowly extending cellulitis with
development of painful ulcerated areas around wire stay sutures at an abdominal operation wound site.
Bacteraemia and metastatic abscesses are seen in non-treated subjects.
Anaerobic cellulitis is a necrotising infection of devitalised subcutaneous tissues, and does not involve
deep fascia to a great extent. Myositis is uncommon in contrast to gas gangrene. The typical clinical
findings are crepitation, local pain, tissue swelling, and a thin, dark foul-smelling exudation from the wound.
Clinical signs of systemic toxicity are not prominent.

Infecting organisms

Likely infecting organisms in erysipelas and cellulitis: Group A streptococci (Streptococcus pyogenes), also
Staphylococcus aureus in cellulitis.
Microaerophilic and anaerobes streptococci, Staphylococcus aureus and Gram-negative bacilli can be
isolated in aspirates of the lesions in progressive bacterial synergistic gangrene.
Clostridium species, usually Clostridium perfringens, are involved in anaerobic cellulitis.

Treatment

Erysipelas and cellulitis should be treated with penicillinase-resistant penicillin. Cefazolin or clindamycin are
alternatives in patients allergic to penicillin.
In anaerobic cellulitis surgical exploration and debridement are essential, with amoxicillin-clavulanate and
clindamycin as initial antimicrobial therapy.

Necrotising fasciitis

Clinical signs and diagnosis

Necrotising fasciitis is a rare but severe infection with a mortality rate ranging from 20% to 47%. The
infection involves the subcutaneous soft tissues and extends to the deep fascia. It is accompanied by
clinical signs of sepsis, septic shock, and multi-organ dysfunction. Serum creatinine phosphokinase levels
may be markedly elevated as an indication of myositis.
The affected area is initially erythematous, swollen, hot, shiny, tender, Necrotising fasciitis should be
and extremely painful. This area rapidly expands, the skin colour suggested when there is a
changes from dark red to blue-grey, and vesicles and violaceous bullae great discrepancy between
appear. Anaesthesia develops as a consequence of thrombosis of small local findings and the patient's
blood vessels and destruction of superficial nerves. Subcutaneous gas presentation with extreme pain
may be present.
Gram-stained smears of exudates, blood cultures, biopsy specimens
(dermis, infected subcutaneous tissue, fascia, underlying muscle)
should be taken.
Fournier's gangrene is a form of necrotising fasciitis occurring around
the scrotum (in males) or genitals (in females), with possible extension
and involvement of the perineum, penis, and abdominal wall.

Infecting organisms

Type I: Anaerobic species (Peptostreptococcus, Bacteroides spp.) are isolated in combination with
facultative aerobic species such as streptococci (other than group A) and Enterobacteriaceae (E. coli,
Klebsiella, Enterobacter, Proteus)
Type II (haemolytic streptococcal gangrene): Group A streptococci is isolated alone or in combination
with other species, commonly Staphylococcus aureus.

Treatment

Necrotising fasciitis is a surgical emergency. Prompt surgical exploration with extensive debridement and
removal of necrotic tissue down to deep fascia and muscle is essential. The incisions should be made
extensively until normal fascia is found, and the wound should be left open. A second look 24 hours later
should ensure the adequacy of the initial debridement.
Initial antibiotic therapy is based on the bacteria likely to be involved and includes a combination of
amoxicillin-clavulanate and clindamycin, or ceftriaxone and clindamycin.
Use of intravenous immunoglobulins is controversial in type II necrotising fasciitis.

NOTE In case of infection with group A streptococci, look for colonisation in family
members by carrying out a nasopharyngeal swab.

Gas gangrene

Gas gangrene usually follows muscle injury with dirty contamination and is a rapidly progressive, life-
threatening infection with an acute onset. Intense pain, local oedema and tenderness, a characteristic
bronze discolouration and bullae of the skin with preceding deep injury or surgery, foul odour of the wound
and drainage (gas bubbles may be visible), and the association with shock should alert the clinician.
Crepitation is usually present, and X-ray proves the gaseous dissection of muscles and fascia.

Infecting organisms

Clostridium perfringens is the organism of the clostridial species most commonly isolated from the wound
exudates or aspirates. Other organisms like Gram-negative bacilli (E. coli, Enterobacter) and enterococci
may also be present, especially in grossly contaminated wounds.

Treatment

As in necrotising fasciitis, gas gangrene is a surgical emergency. It requires extensive debridement, with
excision of the involved muscles and fasciotomies for decompression and drainage. Penicillin in
combination with clindamycin is the antibiotic therapy of choice for Clostridium perfringens. If both Gram-
negative and Gram-positive bacilli are seen in the Gram-stained smears of the wound, ceftriaxone or
ciprofloxacin should be added. Hyperbaric oxygen therapy may be helpful in some patients, but actually
there is not sufficient evidence to recommend the therapy as adjunctive treatment.

Which organism can produce serious soft tissue infection without pus?

Nosocomial infections

Fever is a common problem in the ICU patient, as you will see in the Pyrexia module referenced below.
These infections are potentially avoidable however, as indicated in the module on Infection control
strategies. For more information see the PACT modules on Pyrexia and Infection control strategies

The main diagnostic dilemma is to distinguish between fevers of infectious and non-infectious origin, to
distinguish infection from colonisation, and to determine the site of, and the pathogen involved in,
infection.
In the intensive care environment the patient's oropharynx, respiratory secretions, skin, wounds, urinary
catheter, and digestive tract are rapidly colonised by hospital-acquired bacteria.
By using a systematic clinical approach in eliminating non-infectious disorders, the clinician can focus
on the common areas where nosocomial infections may originate:

Respiratory sources
Intravascular catheter-related infections
Gastrointestinal sources
Urogenital sources
Surgical infections
Fungal infections
Respiratory sources

Nosocomial pneumonia

The most common hospital-acquired infection, at 46.9%, is nosocomial pneumonia. You will find further
information in the PACT module on Pyrexia .
Microaspiration of upper airway secretions colonised with pathogenic bacteria is an important step in
the pathogenesis of nosocomial pneumonia. Supine position, impaired host defences (disruption of
natural anatomic barriers by an endotracheal tube or a nasogastric tube, pooling and leakage of
contaminated secretions around the endotracheal tube cuff, alkalinisation of the normally acid gastric
environment), and contaminated respiratory therapy equipment facilitate antimicrobial entry to the lung.

NOTE Faecal–oral cross-infection by the hands of healthcare professionals may also be


a potential pathway for colonisation of the oropharynx in ventilated patients.

Polymicrobial aetiology is frequent, but in patients with early-onset nosocomial pneumonia the
causative agents are principally Haemophilus influenzae, Streptococcus pneumoniae, methicillin-
sensitive Staphylococcus aureus (MSSA), and non-resistant enteric Gram-negative bacilli (Escherichia
coli, Klebsiella spp., Proteus spp., Enterobacter spp., and Serratia marcescens). In addition to the
above-mentioned core organisms, Pseudomonas aeruginosa, Acinetobacter spp., methicillin-resistant
Staphylococcus aureus, and multiresistant Gram-negative bacilli are more frequent in late-onset
nosocomial pneumonia. The incidence of multiresistant pathogens is linked to previous antimicrobial
therapy, and especially to prior use of broad-spectrum drugs, but also to local factors and co-
morbidities.

T HINK Be aware of the epidemiologic data in your own treatment centre.

Anaerobic bacteria, Legionella spp., fungi, and viruses (especially cytomegalovirus (CMV)) are not
considered to be common in nosocomial pneumonia, but should be considered in apparently culture
negative patients. For more information see the PACT module on Respiratory failure .

Clinical signs

Usually, the diagnosis of pneumonia in the critically ill, mechanically ventilated patient is based on
systemic signs of infection, purulent-looking pulmonary secretions with new or worsening infiltrates
seen on the chest X-ray, and bacteriological evidence of pulmonary parenchymal infection. The
diagnosis remains a difficult dilemma for the clinician, however, as this combination of signs is shared
by a variety of other conditions such as ARDS. In addition, microscopy evaluation and culture of
tracheal secretions are frequently inconclusive because of the bacterial colonisation of the
tracheobronchial tree.

Diagnosis

Invasive bronchoscopic methods such as the use of the PSB technique or the BAL, which has a slightly
higher sensitivity, remain controversial diagnostic procedures, but have the advantages of airway
visualisation and direct access to the lower respiratory airways for sampling at the site of lung
inflammation. Blind sampling of distal airways using a protected endobronchial catheter or a mini-BAL
provide acceptable results with a specificity of 86-100% and a sensitivity of 78%. Mixed cultures, which
are common in nosocomial pneumonia, may be considered negative if individual species are less than
104 CFU/ml (PSB, BAL, mini-BAL).

Nosocomial sinusitis

Intubation is the major risk factor for development of nosocomial sinusitis in ICU patients. For further
information, see the PACT module on Pyrexia .
Nasogastric tubes may cause nosocomial sinusitis, accounting for as many as 16% of cases with fever
of unknown origin in the ICU.

Intravascular catheter-related infections

Infection associated with intravascular catheters is an important and serious complication in critically ill
patients. See the table below in the electronic version and the PACT module on Pyrexia for further
information.

Risk
factors for
catheter-
related
infections

Gastrointestinal sources

Antibiotic-associated colitis

Antibiotics, especially clindamycin, broad-spectrum β-lactam antibiotics, and cephalosporins, are the
most important precipitating causes of Clostridium difficile pseudomembranous colitis, a toxin-related
acute inflammation of the colonic mucosa. For information on antibiotic-associated diarrhoea see the
references on the next screen.

NOTE Hospital-acquired diarrhoea may also be of benign, non-infectious origin, caused


by medication-induced changes of the colonic flora. The treatment consists
simply in discontinuing antibiotic therapy and in replacing fluid and electrolyte
losses. Diarrhoea in the critically ill patient may also be due to the osmotic effect
of rapid administration of enteric feeds, lactose intolerance, or mucosal damage
due to ischaemia.

Urogenital sources

Aetiology

Nosocomial urinary tract infections are related to urethral catheterisation or invasive urinary tract
procedures, and include as risk factors the duration of catheterisation, diabetes mellitus and renal
failure, and the absence of systemic antibiotic treatment.
The colonisation by micro-organisms from the colonic flora along the urinary catheter is inevitable, and
bacteriuria, defined as a quantitative culture >105 CFU/ml, is common in ICU patients, although in most
episodes it is asymptomatic. Bacteraemia is rare, and requires antibiotic treatment and removal or
exchange of the catheter.
Treatment for asymptomatic bacteriuria is only recommended in patients with kidney stones or with
urinary tract obstruction, and if bacteriuria follows urinary tract manipulations or surgery.

NOTE A closed sterile drainage system will delay but not prevent catheter infection.
Suprapubic catheters have been demonstrated to be associated with a lower risk
of urinary tract infections, and may also reduce the risk of local complications
(prostatitis, epididymitis, urethral stricture).

Surgical infections

Patients with multiple injuries and patients after surgical interventions who require care in the intensive
care unit are at increased risk of developing infectious complications. Additional risk factors include
shock, coma, mechanical ventilation, a prolonged ICU stay, advanced age, co-morbidities,
immunocompromised conditions, renal failure, invasive monitoring, and previous antibiotic therapy.
On the other hand, infections in critically ill patients with multiple injuries and/or after surgical
interventions may be particularly difficult to diagnose because clinical signs of sepsis are often not
reliable, informative history is often not obtainable, and findings on physical examination are masked by
a decreased level of consciousness.
Because infectious complications occur at the site of injury, at the site where invasive procedures and
operations have been performed, and at distant nosocomial sites, the clinician should systematically
evaluate each potential site in an effort to define the focus.
Ventriculitis is a serious complication in neurosurgical patients who need a ventriculostomy catheter
for continuous intracranial pressure monitoring and external CSF drainage. CSF leaks and duration of
catheterisation longer than four to five days are the main independent risk factors, and Gram-positive
micro-organisms are the typical pathogens. In a recent review of ventriculostomy-related infections
(VRI) in critically ill patients, infection rate was 9% and it only prolonged ICU stay but had no effect on
survival.

In diagnostics, an increasing CSF cell count and a positive Gram stain should lead to the suspicion of
bacteriological drainage contamination. In patients with a subarachnoid catheter or a subarachnoid
bolt, the risk of an infection seems to be much lower.
Sternum infections and mediastinitis are feared complications after cardiovascular surgery. Obesity,
smoking, a prolonged ICU stay, an infection at another site, and the use of two internal mammary
arteries are recognised risk factors, but pathophysiologically, the aspect and postoperative stability of
the bone and the surgeon' experience are also important. Unusually severe local pain and instability of
the bone, coupled with erythema, hyperaemia, tissue oedema, and liquid collections on the CT scan,
are the leading clinical symptoms and signs.
Early prosthetic valve endocarditis is a rare complication, associated with a mortality exceeding
30%. Clinically, new or changing cardiac murmurs, regurgitant murmurs in association with
atrioventricular conduction abnormalities or congestive heart failure may be apparent, and the
transoesophageal echocardiography may show evidence for valvular dehiscence, paravalvular leaks,
or myocardial abscess formation. The hallmark in diagnosis, however, is bacteraemia. After valve
replacement surgery, the prosthetic valve is not yet endothelialised in the immediate postoperative
period and therefore quite susceptible to microbial colonisation, originating from tissue surfaces,
contaminated intravascular catheters and devices, or postoperative infections at extracardiac sites.
In the therapeutic approach of non-valvular intravascular device-related infections (pacemakers
and implantable cardioverter defibrillators, endovascular grafts and stents, left ventricular assist
devices) a combination of antibiotic therapy and surgical intervention is recommended. Removal of the
infected implant is often needed for cure.
In critically ill patients who have undergone abdominal surgery, deterioration of organ system function
and peritonitis may arise as complications of a surgical procedure. See the PACT module on
Abdominal problems for more information .
The skin and soft tissue in the patient with multiple injuries or burns, or in the post-surgical critically ill
patient, are highly vulnerable to both injury and compromised wound healing.
Wound and soft tissue infections can occur at any site where the skin barrier has been breached,
and where local hypoperfusion and tissue hypoxia have caused tissue ischaemia and necrosis,
promoting further excessive wound contamination. The clinician should be alerted by wounds that are
unusually tender to palpation, a change in wound appearance or character, the presence of cloudy
secretions or purulence within a surgical incision, and non-healing of a bone after an open fracture.

NOTE Nosocomial pathogens, as well as antibiotic-resistant bacteria after previous or


prophylactic antibiotic therapy, may have become the dominant colonising flora in
surgical wound infections.

Typical pathogens
in nosocomial
surgical infections

For more information, see the following references.

Fungal infections

The incidence of nosocomial systemic fungal infections, caused by Candida spp. in nearly 80% of
cases (mostly Candida albicans, but also non-albicans species such as Candida tropicalis, Candida
parapsilosis, Candida glabrata and Candida krusei), accounts for 10-15% (17% in the EPIC study) of
all nosocomial infections, and the mortality rate is as high as 60%.
In the paediatric study by Richards et al. of infections in the PICUs in the USA, C. albicans accounted
for 5.5% and all Candida species for about 10% of BSI, 2.4% of pneumonias and 21% of urinary tract
infections.
Candida species are constituents of the normal flora of the human oropharyngeal, gastrointestinal, and
genitourinary tracts. In hospitalised patients receiving broad-spectrum antibiotics, colonisation of
mucosal surfaces is favoured, and insufficiencies in host defences with breakdown of mucosal barriers
are responsible for haematologic dissemination. However, not all patients colonised with Candida spp.
will become infected.
Apart from colonisation and broad-spectrum antibiotics, major risk factors include an indwelling central
venous catheter, total parenteral nutrition, compromised immune status, burns, major abdominal
surgery, major trauma, diarrhoea or ileus, haemodialysis or steroidal treatment.
Given the non-specificity of clinical features, the high mortality rate, and the benefit of an early
treatment, it is thus important for the clinician to know and to identify the risk factors for invasive
candidiasis, and to have a high index of suspicion in critically ill patients. All patients with risk factors,
persistent occult infection, and/or inexplicable worsening of organ function should therefore be
evaluated for invasive fungaemia, and undergo an ophthalmologic examination for endophthalmitis.
What are the main risk factors for fungal infection in patients in the ICU?

Rapid diagnostic techniques

Latex agglutination test

Antibodies to Mycoplasma pneumoniae in serum are detected by agglutination of gelatin particles


coated with antigens from the organism. A positive result may be obtained five days after the start of
illness.

Direct antigen detection

Clinical, analytical, and radiological techniques cannot distinguish pneumonia caused by Legionella
and non-Legionella aerobic organisms. For this reason, specialised laboratory tests are necessary to
establish the diagnosis. The Legionella urinary antigen test is a rapid test that detects antigens of L.
pneumophila in urine in as little as 15 minutes. Sensitivity is around 50% and specificity is near 100%.
The test is not influenced by previous use of appropriate antibiotics and can remain positive for several
weeks after infection has occurred. The only drawback of the test is that it only detects L. pneumophila
serogroup 1, which accounts for about 70% of cases.

Clostridium difficile toxin detection

The enzyme immunoassay (EIA) test for toxin A and B provides a result within 2-3 hours, but the tissue
culture toxin assay is a more sensitive test for C. difficile cytotoxin B and is the gold standard for
diagnosis.

Polymerase chain reaction

Polymerase chain reaction (PCR) is useful for detecting organisms not easily cultured, e.g.
Mycobacterium tuberculosis, Toxoplasma gondii, and Chlamydia sp. PCR-based assays have also
been used for the detection of Legionella in clinical samples such as urine and serum. PCR is resource
intensive and expensive, however, so is not used as much in bacterial diagnosis as in virological
diagnosis, e.g. HIV, hepatitis C, herpes simplex virus (HSV).

What is the difference between specificity and sensitivity of microbiological tests? What is the
relevance for practice?

Biological markers of infection

C-reactive protein (CRP) and procalcitonin (PCT) are straightforward to assay and are fairly accurate in
differentiating infection from other causes of inflammatory response. They are also more responsive to
the progression of infection than erythrocyte sedimentation rate (ESR).
Research is accumulating on PCT, and it may prove to be useful, particularly if serial assays are made.
In a recent study, PCT levels of 1.5 ng/ml or more had a sensitivity of 100% and a specificity of 72% in
identifying probable or definite sepsis. CRP of 50 mg/l or more had a sensitivity of 98.5% and a
specificity of 75%. PCT has correlated better with TNF-α and IL-6 than CRP and other clinical
parameters such as leukocyte count or fever. A low PCT level (<0.4 ng/ml) excludes bacteraemia as a
diagnosis.

Is a diagnostic procedure needed before starting antibiotic therapy?


In this section we will consider only microbiological diagnostic procedures. The indication for such a
procedure depends on the following factors:

The degree of urgency for starting antibiotics (discussed further in Task 2 )


The patient's condition, stable or unstable, determining the possibility of invasive procedures
The site of infection
Previous antibiotic therapy, which can result in false negative results
The potential impact of the procedure on timing of antibiotic treatment

PRACTICE It is almost always possible to perform some diagnostic procedures before starting
antibiotics. In certain situations it is important to carry out diagnostic procedures before
initiating antibiotic therapy.

Community-acquired meningitis

Perform lumbar puncture before starting antibiotics. In cases of intracranial Severe infection, however,
lumbar puncture will be delayed and antibiotics are given immediately after blood has been obtained for
culture.

Community-acquired pneumonia

Two blood culture specimens (30-60 min interval between sets), and potentially infected body fluids
including pleural fluid, joint fluid, and CSF should be taken before starting antibiotics. Sputum
examination is a relatively simple and inexpensive procedure. The usefulness of fibre-optic
bronchoscopy is debatable and this procedure may be dangerous in hypoxaemic non-intubated
patients.

Ventilator-associated pneumonia

Unlike community-acquired pneumonia, it may be difficult to determine whether pneumonia has


developed in a patient on mechanical ventilation. Moreover, many different pathogens may be
implicated. If microbiological diagnosis of ventilator-associated pneumonia (VAP) is made by using
quantitative cultures (BAL or PSB), the procedure should be performed before starting or changing
antibiotics. Indeed, recently initiated therapy may decrease the sensitivity of invasive diagnostic
methods.

If you suspect VAP and you decide to take respiratory specimens for quantitative
cultures, do not initiate new antibiotic treatment before the procedure.

Necrotising fasciitis

Before starting antibiotics, draw a blood culture, and as soon as possible ask for Gram-stained smears
of the exudate.

Is it necessary and cost effective to perform a cranial CT scan before lumbar puncture in a
patient with suspected bacterial meningitis? Explain your answer.

Gram stain: advantages, limitations and pitfalls


Direct examination of specimens with Gram stain is the first step in microbiological diagnosis. This
technique should be performed in all samples obtained from normally sterile fluids and from drained
collections.
Gram stain may offer useful guidance in the choice of initial empirical antibiotic treatment. If Gram stain
is positive with a single species, it may be possible in some circumstances to use an initial antibiotic
regimen with a narrower spectrum. This latter statement includes both Gram-negative and Gram-
positive infections. For example, the recognition in sputum of lancet-shaped Gram-positive diplococci
suggesting S. pneumoniae may help physicians to select appropriate antibiotics in a patient with
community-acquired pneumonia.

Limitations and pitfalls of Gram stain

Not always available from the microbiology laboratory, especially at night.


Sometimes may be difficult to interpret; for example, Acinetobacter baumannii strains may appear as
coccoid cells quite similar to Neisseria spp., or because of Gram stain variability, as staphylococci.
In polymicrobial infections such as abdominal infections, hepatic infections, cerebral abscess, or
aspiration pneumonia, direct examination of samples is less useful since a broad-spectrum antibiotic
regimen is always recommended.
Gram stain may be negative despite the presence of true infection, especially if the inoculum is weak.
In patients already treated with appropriate antibiotics, the significance of positive Gram stain is
uncertain, and may correspond to 'dead bacteria' (negative cultures).

2/ DEVELOPING A THERAPEUTIC STRATEGY


It is not always possible or easy to determine the source of an infection. Once the source is found or
suspected there are a number of issues which need to be addressed.

Source control in severe sepsis


The potential role of source control measures should be discussed for all patients with severe sepsis.
Patients can be divided into three groups according to the site and the type of infection:

Patients in whom the need for source control is obvious, requiring surgical procedure or drainage in
all cases; i.e. a patient with diffuse peritonitis from a perforated lesion in the gastrointestinal tract or
with necrotising fasciitis, or with a pleural empyema.
Patients in whom the source of infection is an infected foreign body. In some cases, the source of
infection can be easily removed and changed, such as a urinary catheter or a vascular catheter. The
solution may be more difficult when the foreign body is an articular or cardiac prosthesis or a vascular
graft.
Patients in whom there is obviously no need for a surgical procedure. However, in order to decrease
bacterial inoculum size, some additional measures may be useful, such as tracheal suctioning and
physiotherapy or prone position in bacterial pneumonia occurring in patients on mechanical
ventilation.

Timing of source control interventions

Optimal timing of intervention is difficult to define and should be discussed


with the surgeon and the radiologist. The following steps are proposed:

Identify situations in which source control is urgent' for example,


necrotising soft tissue infection, diffuse peritonitis, sepsis caused by
urinary tract obstruction.
Try to appropriately stabilise the patient, since aggressive and rapid
resuscitation can reduce the operative risk.
Perform the diagnostic procedure according to the site of infection and the
patient's condition.
Decide whether to operate.

ANECDOTE A 42-year-old male was treated with a combination of cefotaxime and vancomycin for
suspected pneumococcal meningitis. On day two, a second lumbar puncture was
performed. CSF Gram stain was still positive, although the strain was fully susceptible to
penicillin G. Despite persistent positive Gram stain, CSF cultures were negative and the
patient fully recovered after treatment with amoxicillin.

Is there an absolute urgency to start antibiotics?


Outcome of severe infection is mainly dependent on two parameters: virulence of the pathogen and
host susceptibility. The latter is influenced by several factors, such as immune status, co-morbidities,
and genetic polymorphism. In some individuals, an enhanced inflammatory response may lead to
multiple organ failure despite early administration of antibiotics. Prompt antimicrobial treatment may be
useful to reduce morbidity. For example, early appropriate antibiotics may prevent the development of
metastatic localisations (osteoarticular or cardiac) in patients with S. aureus bacteraemia.
In a recent Canadian study to determine factors which predicted in-hospital mortality amongst patients
who required hospitalisation for community-acquired pneumonia, it was concluded that early mortality
is not affected by timing or type of antibiotic therapy, but late mortality is influenced by the type of
antibiotic therapy.

Patients can be classified into one of the following three categories according to the degree of urgency
for starting antibiotic treatment:

Delayed initiation of appropriate antibiotic therapy may increase mortality and


morbidity.

Antibiotic treatment is an emergency

The following infections may be classified in this group:

Severe sepsis or septic shock.


Infections known to have a fulminant course, such as meningococcaemia. Antibiotics should even be
administered before hospital admission, if possible.
Bacterial meningitis: antibiotics should be given if a diagnosis of bacterial meningitis is suspected.
Infection can be confirmed by the presence of purulent or turbid cerebrospinal fluid from lumbar
puncture.
Sepsis in a splenectomised patient.
Severe sepsis or clinically documented infection in neutropenic patients, such as pneumonia or
abdominal or soft tissue infection.

Antibiotic treatment is urgent but not an emergency

The patient belongs to neither group 1 nor group 3. This is the case for a significant proportion of
obvious or suspected infections in critically ill patients, either requiring ICU admission or occurring in
patients already hospitalised in such a unit. In most instances, antibiotic treatment should be started as
soon as possible, and no later than six to eight hours. If the infection is obvious or the patient is
unstable, there is no advantage in waiting.

The patient has fever but is stable

This situation is not infrequent in ICU patients on mechanical ventilation. A wide range of biological
processes, some infectious and many non-infectious, can cause fever. Physicians should be
encouraged to perform the appropriate diagnostic procedures according to the suspected source of
infection. Antibiotics may then be started:

On the same day if there is clear evidence of infection

OR

After 24 hr (or more), pending the results of cultures.

One good example is suspected VAP. Following BAL or PSB, if the patient's condition is stable, it is
wise to wait 24 hours for the results of quantitative cultures before deciding whether to initiate antibiotic
treatment.

In the next five patients you see, use the above classification prior to starting
antibiotics. Reflect on its usefulness.

What is the relationship between outcome and timing of antibiotics in severe community-
acquired pneumonia?

3/ STARTING INITIAL ANTIBIOTIC MANAGEMENT

Optimal antibiotic therapy?


Difficulty in identifying and confirming an infecting organism means In most cases of infection in
that empiric antibiotic treatment is the normal practice rather than the ICU the actual causative
the exception in the ICU. Selecting appropriate empiric pathogen is rarely determined
antimicrobial therapy should maximise clinical impact while with certainty
minimising toxicity and the development of resistance. In practice,
this involves prescribing antimicrobial therapy when it may be
beneficial to the patient, targeting therapy to identified pathogens,
and using the appropriate drug, dose, and duration of treatment.
Empiric antibiotic strategies should be developed with care, taking
into account the most prevalent and virulent pathogens, their local
resistance profile, as well as the patient. Historical practices are
being challenged, such as long duration of therapy and selection of
antibiotic based on general clinical practice guidelines, and new
practices are being introduced.

To choose the appropriate treatment for the individual patient, consider:

Relative effectiveness of the antibiotic


Choice of agent if there is organ dysfunction or the patient is elderly
Contraindications, warnings and precautions (toxicity considerations) relevant to the patient
Drug penetration to the site of infection
Convenience of route of administration and dosage for the patient

Patients with COPD or one week of ventilation should receive combination therapy.

Advantages and
disadvantages of
combination therapy

When is antibiotic monotherapy not recommended?

In suspected nosocomial pneumonia, empiric antibiotic treatment should not be delayed, as significant
morbidity and mortality are attached, and patient survival may improve if pneumonia is correctly
diagnosed and treated.
In early-onset pneumonia a second-generation or third-generation cephalosporin, a β-lactam/β-
lactamase inhibitor combination, or alternatively a fluoroquinolone should be administered. In late-onset
pneumonia the recommended treatment includes an aminoglycoside (anti-pseudomonas
fluoroquinolone) plus a broad-spectrum antibiotic with anti-pseudomonal activity, and a glycopeptide (or
oxazolidinone) if infection with MRSA is strongly suspected. Based on the clinical response and the
results of cultures available at day two to day three, the antibiotic therapy may be de-escalated in the
further course.

Understanding underlying pharmacodynamics


Pharmacodynamic characteristics of antibiotics determine how the antibiotic exerts its antimicrobial
effect. Some antibiotics are consistently bactericidal only so long as the concentration is above the
minimum inhibitory concentration (MIC) of the target organism (time-dependent killing), while others are
effective in relation to the peak concentration achieved at the site of infection (concentration-dependent
killing).

Penetration

Antibiotic treatment failure may be due to poor penetration of the antibiotic into infected tissues.
MRSA is the second most frequently isolated pathogen in patients who die from pneumonia. Treatment
options for this pathogen are limited. Vancomycin has demonstrated poor effectiveness in treating
pneumonia caused by MRSA. This may be due to poor penetration of vancomycin into the alveolar
space. Despite its better penetration, there have also been failures of treatment with teicoplanin. For
further information about teicoplanin see the following reference.

Administering vancomycin as a continuous infusion and maintaining constant concentration in serum of


four to five times the MIC for the infecting organism may be the ideal way to deliver vancomycin for
serious infection. You can find more on antibiotic management programmes in the following references.

Antibiotics with time-dependent killing


Glycopeptides and β-lactam antibiotics should be administered regularly so that the antibiotic
concentration is above the microbial MIC for as long as possible. In addition to frequent dosing this
may be achieved by:

Administering another drug that interferes with antibiotic elimination


Replacing the drug with a therapeutically equivalent antibiotic with a longer half life
Administering the antibiotic by continuous i.v. infusion

Antibiotics with concentration-dependent killing

Aminoglycosides (e.g. gentamicin) and fluoroquinolones (e.g. ciprofloxacin, levofloxacin) have


concentration-dependent killing activity and so are given at high doses at less frequent intervals.
However, this may have an impact on toxicity. Aminoglycosides eradicate organisms best at 10-12
times the MIC of the micro-organisms, so a single daily dose may produce a better clinical response
than the same amount of drug given in divided doses. Toxicity correlates with tissue accumulation and
not with peak serum concentration.
High concentrations of fluoroquinolones are associated with seizures and other serious CNS
complications, so high doses cannot be given. In this case the ratio of the area under the curve (AUC)
to MIC (AUIC) may be a good predictor of clinical response.

Failure to monitor aminoglycoside levels or act when they become toxic is a


common reason for negligence litigation.

Post-antibiotic effect

This is defined as a persistent suppression of bacterial growth even after the serum antibiotic
concentration falls below the MIC of the target organism. Most antibiotics have a post-antibiotic effect
(PAE) against Gram-positive bacteria. Aminoglycosides and fluoroquinolones have a PAE against
Gram-negative bacteria. You can find out more about antibiotic pharmacodynamics in the following
references.

What empiric antibiotics are generally used for hospital-acquired pneumonia?

Microbial resistance

Several management practices are being investigated which aim to help reduce the development of
microbial resistance and improve outcomes. These include:

Microbial surveillance

Local, national and international surveillance programmes aim to control the spread of resistant
bacteria. Surveillance of isolates obtained from critically ill patients aims to anticipate the sensitivity of
pathogens so that appropriate antibiotic use can improve outcomes. Surveillance initiatives include
molecular tracking of major epidemic clones of multiple-resistant pathogens and evaluation of the
ecological impact of antibiotic policies.

Examine the resistance patterns for all pathogens from patients admitted to the
ICU over one week. How can collection of this information help future patient
treatment? How many are caused by cross infection and how many are present on
admission?
Observe how many times staff wash their hands when attending to a patient
during a 20-minute period. Count the number of sinks and bottles of alcohol hand
rub.

Hospital formularies that restrict the use of antibiotics have been successful in
dealing with outbreaks of infection with antibiotic-resistant bacteria, particularly
in conjunction with infection control practices. However, they cannot be
considered as an alternative to judicious use of antibiotics, since resistance is
likely to develop to the antibiotics that are not restricted.

Antibiotic rotation

PRACTICE In an attempt to limit bacterial resistance, a class of antibiotics or a specific antibiotic is


withdrawn from use for a defined time period and then reintroduced. Antibiotic rotation
can reduce the incidence of antibiotic-resistant bacterial infections and may reduce
overall infectious mortality, without increasing antibiotic costs.

ANECDOTE First-line antibiotics can affect the microbial flora of the ICU. When cephalosporin use
was severely restricted in Denmark, the prevalence of MRSA was dramatically reduced
and remains low.

How does antibiotic rotation reduce microbial resistance?

PRACTICE Every ICU should have an antibiotic strategy or programme in place that monitors
antibiotic utilisation and effectiveness. This is the only way in which the impact of
interventions aimed at improving the use of antibiotics can be evaluated at the local
level.

Is antibiotic prophylaxis for respiratory tract infections an effective strategy in the intensive care
unit? Give reasons for your answer.

4/ REASSESSING INITIAL MANAGEMENT


Appropriate empiric antibiotic treatment, started at the first signs of infection, can reduce infection rates
and improve survival. Once infecting organisms have been identified, or if the patient is not responding
to treatment, it is necessary to reassess initial empiric management and make appropriate changes. In
this Task, we will focus on the importance and methods of reassessment, reasons for clinical failure of
antimicrobial therapy, and reasons for changing antibiotic treatment.

Reassess the patient


Reassessment of initial management is necessary:

On day two to three to decide if antibiotic treatment should be stopped (because there is no bacterial
infection) or changed.
On day seven to ten to decide if antibiotic treatment should be continued.
At any time if failure is suspected.
If there is apparent clinical failure there are three important areas of investigation:

Examine the patient. Pay attention to intravascular catheters and surgical wound sites.
Discuss the apparent failure with the microbiologist. Check the results of susceptibility tests when
available.
Obtain new specimens for microbiological analysis according to clinical assessment rather than
laboratory requests.

What tests should be carried out in a patient whose community-acquired pneumonia is not
responding to antibiotic therapy and in whom a definitive diagnosis is not established?

Failure of initial management

As with other anti-infective therapy, effective source control should be rethought if at least one of the
following situations is present:

No resolution of clinical signs of sepsis or organ dysfunction


Persistent positive cultures (discuss colonisation vs infection)
No evidence of reversal of metabolic disorders
Insufficient progression of wound healing
Radiographic evidence (chest X-ray, ultrasonography, CT scan) of an infectious focus

PRACTICE There are six questions you should answer when initial management fails:

Was the initial anti-infective treatment appropriate?


Is there a possible focus of infection that has not been eradicated (catheter to remove,
abscess to drain,...)?
Has the initial pathogen developed resistance to the antibiotics?
Is the pathogen non-bacterial: e.g. a fungus such as Aspergillus spp. or Candida spp.?
Has the patient developed nosocomial infection?
Is the fever due to non-infectious aetiologies?

In which patients is a fungal infection likely?

ANECDOTE A 34-year-old man was admitted to the ICU with severe pneumococcal pneumonia
requiring mechanical ventilation. He was treated with amoxicillin. Despite improvement
in gas exchange, fever was still present on day five. Fever settled within 48 hours of
change of antibiotic to clarithromycin. No pleural empyema was found.

What are the usual suspected bacterial pathogens in hospital-acquired pneumonia?

Change initial therapy

There are five reasons to change initial therapy:

The pathogen is not susceptible to the chosen regimen.


The infection is polymicrobial and not all of the pathogens are adequately covered.
Combination therapy was used but since the pathogen is fully susceptible, monotherapy may be
adequate.
Monotherapy was used but the bacterium is not fully susceptible (i.e. P. aeruginosa) and thus
combination therapy may be preferable.
Antibiotics with a narrower antimicrobial spectrum are active, enabling cessation of broad-spectrum
antibiotics: a de-escalation strategy.

The de-escalation strategy is proposed to curtail the development of antimicrobial


resistance and to reduce the costs, for example by using 'older' antibiotics. Use
glycopeptides only when necessary.

Length of antibiotic therapy


The antibiotic course needs to be long enough to prevent the risk of Make sure you achieve the
relapse caused by insufficient duration and yet short enough to optimal duration
avoid needless antibiotic administration that encourages
antimicrobial resistance.

There is no standard duration of antibiotic treatment. Length of treatment should be determined by


consideration of a number of factors:

What is the immune status of the patient?


What is the involved pathogen?
What is the site of infection?
Is the clinical response to appropriate antibiotic treatment favourable?
Is the patient still exposed to the risk: i.e. if the patient was treated for VAP, is he still on mechanical
ventilation?
Was the focus of infection removed?
Does the patient have a prosthetic device (endovascular, osteo-articular)?

A short duration treatment is probably effective when:

The patient is immunocompetent


The pathogen is fully susceptible to antibiotics
There is no internal device
The antibiotic is thought to readily diffuse into the site of infection (i.e. absence of deep-seated
infection such as endocarditis)

Examples of
suggested
lengths of
antibiotic
treatment

Mechanisms of resistance
The incidence of nosocomial infections due to resistant organisms is increasing. Infections due to
resistant organisms are particularly important in the ICU because they are associated with higher
morbidity, longer ICU stays, and higher healthcare costs. The emergence of resistance is due to:

Increased use and misuse of antimicrobial agents, particularly repeated exposure to sub-optimal
antibiotic concentrations
An increasing number of invasive devices and procedures
Increasingly susceptible patients, e.g. immunocompromised patients or those with severe underlying
illness
Lapses in infection control measures

The incidence of bacterial resistance in ICU patients has been reported and discussed in the EPIC
study and the SENTRY antimicrobial surveillance programme. See the references below for further
details.

Development of resistance to antibiotics is a possible cause of treatment failure. It


may occur more readily with certain pathogens and is favoured by high inoculum.

Development of resistance

There are three main mechanisms by which bacterial resistance develops:

Acquisition of new genes via transposons or horizontal gene transfer


Selection of resistant variants
Selection of naturally resistant strains

Development of resistance to specific antibiotics during treatment may occur in the following situations.

Minimising the risk of resistance

Try to reduce the inoculum by draining an abscess or removing foreign material when feasible.
Use adequate dosages of antibiotics.
Take into account the pharmacodynamic characteristics of antibiotics. For example, the activity of
aminoglycosides is dependent on peak concentrations, therefore aim to achieve high serum peak
concentrations rapidly.
Use combination therapy (for a few days) when P. aeruginosa or group 3 Enterobacteriaceae are
involved. Never use rifampicin, fosfomycin, and even fluoroquinolone alone when treating S. aureus
infection.

Rules

PRACTICE 1. Empirical antibiotic therapy should be started without delay in septic patients.
2. Only antibiotics not prescribed in the previous two weeks should be
considered.
3. MRSA is not expected in absence of previous antibiotic use.

4. The antibiotic regimen should be targeted based on rapidly obtained direct


tests.
5. The antibiotic regimen should be modified based on microbiologic findings.

6. Use a combination of antibiotics when appropriate.


7. Prolonging antibiotic treatment does not prevent recurrence.

5/ APPENDIX

Returning travellers

Food- and waterborne infections such as typhoid fever, cholera, and hepatitis A may result in critical
illnesses. The most common life-threatening infection in returning travellers is malaria tropica, caused
by the intra-erythrocytic protozoa Plasmodium falciparum. Early recognition and focused therapy of
complicated malaria are critical to successful management and outcome. Malaria should always be
considered in a febrile patient returning from a tropical country. Failure of one or more organ systems
(coma and convulsions due to cerebral malaria, circulatory shock, respiratory distress syndrome and
renal failure, severe anaemia and coagulation disorders with bleeding) and metabolic disorders
(acidosis and hypoglycaemia) are serious and life-threatening complications. Diagnosis of malaria is
made by laboratory examination of thin and thick smears of blood. After one negative blood smear,
they should be repeated every 6-12 hours for 48 hours if malaria is still suspected.

Treatment

In the treatment of severe malaria, intravenous quinine or quinidine should be preferred to parenteral
chloroquine if there is any uncertainty about resistance. Combination with doxycycline, or alternatively
clindamycin, is recommended. Exchange blood transfusion should be considered in high parasitaemia.
Monitoring for toxicity of therapy and treatment response is mandatory. Supportive care also includes
careful attention to fluid balance, as the affected patient is extremely vulnerable to fluid overload.

With the exception of typhoid, supportive rather than antibiotic treatment is needed in infectious
diarrhoeas. Salmonella typhi (other Salmonellae, especially Salmonella paratyphi A and B and
Salmonella choleraesuis, may be responsible for a similar clinical pattern) causes the classical
syndrome of enteric fever with a remittent fever, headache, abdominal pain and tenderness,
constipation or diarrhoea, relative bradycardia, splenomegaly, and leukopenia. Rose spots may be
present. The definitive diagnosis is made by isolating Salmonella typhi or another Salmonella from
blood, stool, urine, or bone marrow. Third-generation cephalosporins or quinolones are used for
therapy, as multiple-drug-resistant isolates unresponsive to chloramphenicol, ampicillin, and
trimethroprim-sulfamethoxazole are becoming increasingly prevalent. All patients with presumed
infectious diarrhoea should be kept in source isolation.
Viral haemorrhagic fevers (Dengue fever virus, Lassa virus, Ebola virus, Marburg virus, Crimean-
Congo haemorrhagic fever (CCHF) virus) are rare, but exclusion of the diagnosis in patients from
endemic areas admitted with fever can cause practical problems for handling of specimens by the
laboratory. Viral haemorrhagic fevers are characterised by fever, headache, respiratory symptoms,
myalgias, wasting, and signs of bleeding (bruises, petechiae, mucous membrane haemorrhage). Cases
must be referred immediately to a regional isolation unit.
Fulminant hepatitis may manifest as acute liver failure with jaundice and hepatic encephalopathy.
Hepatitis B, with co-infection with delta virus, Hepatitis A, Hepatitis E (especially in pregnant women),
yellow fever virus and leptospirosis are the responsible aetiologic agents. For more information see the
PACT module on Acute hepatic failure .

CONCLUSION
The treatment of infection in the critically ill patient requires close cooperation between the intensivist,
the medical/surgical team, and the microbiologist/infectious diseases physician. Diagnosis of the
source of infection may be difficult. Cross infection is a major problem and the ICU may act as a source
of multiresistant organisms for the general wards. Antibiotics must only be used if strictly necessary
and should be effective against the suspected pathogens. Antibiotics should be of high dose and short
duration where possible to limit their environmental impact.

PATIENT CHALLENGES

A 66-year-old female was transferred from the Emergency Department to your Intensive Care Unit
suffering from pyrexia, diarrhoea and dyspnoea. There were no similar cases among the patient's relatives or
friends.

The patient had no history of recent travel or of tick bites.The patient had been taking 10 mg of prednisolone for
four years due to a Horton arteritis and primary biliary cirrhosis. She was well until four days earlier when she
presented to her GP with diarrhoea, chills and fever. There was no history of chest pain. She was given
amoxicillin/clavulanate. On the day of admission she awoke with profuse sweating and dyspnoea at rest.

On admission, the patient's core temperature was 39.5 °C, pulse rate 105 bpm, respiratory rate 28 breaths per
minute and arterial blood pressure 135/80 mmHg. An opacity in the left hemithorax was documented on chest-X
ray. Therapy with levofloxacin was started. Blood cultures were taken and a urinary antigen test was negative for
Streptococcus pneumoniae and Legionella pneumophila.

Learning issues

Food- and waterborne infections

NOTE Mineralocorticoid deficit should be suspected in the presence of prior steroidal therapy and
hydrocortisone prescribed.

Investigations

Twenty-four hours later, the patient was afebrile but the respiratory rate increased to 34 breaths per minute,
diastolic pressure under 60 mmHg, accompanied by blood urea nitrogen >7 mmol/l and she was transferred to
the ICU. Arterial oxygen saturation was 85 per cent with supplementary oxygen by mask. On examination, the
patient was pale and sweaty. Rales were heard at both lung bases and the pulmonary infiltrates were bilateral on
chest X-ray. Antibiotic therapy was changed to cefotaxime and clarithromycin. After an unsuccessful trial of non-
invasive mechanical ventilation, the patient required intubation.

Learning issues

Diagnostic approach

Optimal antibiotic therapy

Changing initial therapy

PACT module on Airway management


What are the likely micro-organisms that might have caused pneumonia in this patient?

Risk factors associated with particular respiratory pathogens

Learning issues

Community-acquired infections

The patient developed severe ARDS, requiring a ventilatory protective strategy. She remained under sedation
and needed parenteral nutrition because of intolerance to enteral nutrition. Pleural drainage was required due to
developing barotrauma.

Ten days later, the patient developed fever of 38.5 °C and pulmonary secretions remained unchanged. WBC
increased to 27 000 cells/mm3. Bilirubin was 1.3 mg/dl. AST 29 U/L, ALT 17 U/L; alkaline phosphatase 320 U/L.
An intravenous catheter was withdrawn and sent to the lab for cultures (with two additional blood cultures).

Learning issues

PACT module on Respiratory failure

PACT module on Mechanical ventilation

PACT module on Nutrition

PACT module on Pyrexia

Nosocomial infections

When should empiric antibiotic therapy for ventilator-associated pneumonia (VAP) have been initiated?

Learning issues

Ventilator-associated pneumonia

Twelve hours later, the patient developed hypotension unresponsive to volume, and oliguria. Norepinephrine
(noradrenaline) was started in response to a diagnosis of septic shock. An abdominal ultrasound was performed
revealing a thickened gall bladder wall, with pericholecystic fluid, subserosal oedema and intramural gas, without
calculi. Percutaneous cholecystostomy was performed under ultrasound guidance. The procedure was followed
by withdrawal of inotropic support. Blood cultures were positive for Escherichia coli. Cefotaxime and
clarithromycin were stopped, and piperacillin/tazobactam initiated.

Learning issues

PACT module on Hypotension

PACT module on Oliguria and anuria

Source control in severe sepsis

PACT module on Sepsis and MODS

Acalculous cholecystitis
PACT module on Abdominal problems

Should antibiotic therapy have been stopped for an interval before taking cultures so that the pathogen could
be more easily identified? Give reasons.
NOTE Delay in prescribing appropriate antibiotic therapy in critically ill patients may result in an
increase in mortality.

Learning issues

Rules for antibiotic therapy


Understanding underlying pharmacodynamics

After a three-week stay, the patient improved but developed generalised muscular weakness.

A percutaneous tracheostomy was performed. Two days later, secretions became purulent, temperature
increased to 38.5 °C and the WBC count increased to 18 000 cells/mm3. A new opacity was documented in the
right lower lobe.

Blood cultures and respiratory specimens were collected for culture. Respiratory secretions were positive for
Pseudomonas aeruginosa.

Learning issues

Investigations

What would be your therapeutic strategy for this patient?

Antibiotic therapy was modified and imipenem and amikacin were started. After 25 days, mechanical ventilation
was stopped and ultimately she was discharged home. Serological tests performed 30 days after admission were
positive for Legionella pneumophila.

Learning issues

Changing initial therapy

NOTE A causative agent is not isolated in as many as 40% of cases of community-acquired


pneumonia.

If direct stains became negative, should antimicrobial therapy be withdrawn in this patient? Give your reasons.

Learning issues

Limitations of Gram stain

Would vancomycin be indicated as empiric therapy for hospital-acquired pneumonia? Why?

Is the presence of glucose non-fermenting Gram-negative bacilli predictable? Give reasons.

Two years later, the same patient was admitted to a local hospital with a one-day history of fever, chills, moderate
headache and pain in the back. Two weeks earlier she had had fever and nasal congestion. She had no
gastrointestinal or urinary tract symptoms.

The physician at the local hospital reported that on arrival she had a temperature of 38.4 °C. Physical
examination revealed normal heart and lung sounds. Glasgow Coma Score was 14. Blood pressure was 145/50
mmHg and her heart rate was regular and 72 beats per minute. No petechiae were seen but meningeal signs
were present.

Learning issues

How to recognise the patient with severe infection


Diagnostic approach

Chest X-ray was normal, and a sinus film showed a discreet shadowing of the right frontal sinus and a slight
congestion of both maxillary sinuses. Blood chemistry showed a WBC count of 15 600 per mm3, haemoglobin of
119 g/l and blood glucose of 6.9 mmol/l (124 mg/dl). Analysis of cerebrospinal fluid (CSF) revealed a WBC count
of 2350 cells (50% lymphocytes), no erythrocytes, albumin of 973 mg/l (normal value 80-260 mg/l), and glucose
levels of 1.05 mmol/l (19 mg/dl). Treatment with high doses of cefotaxime plus ampicillin was started. A
convulsive episode developed, and the patient required intubation and was transferred back to your hospital.

Learning issues

Investigations

NOTE Rapid diagnostic techniques are discussed in Task 2.

What pathogens would you consider?

What is your strategy for antibiotic treatment?

Learning issues

Involvement of particular sites


Developing a therapeutic strategy

What de-escalation therapy could have been used?

Learning issues

Antibiotic de-escalation
You are called to the Emergency Department to evaluate a 61-year-old baker who was admitted on
suspicion of cellulitis of his left lower extremity and a sudden drop in his blood pressure.

The patient's physical examination reveals a body temperature of 38.2 °C, hypotension (90/50 mmHg) with signs
of cutaneous vasoconstriction, irregular tachycardia (145 b/min), and tachypnoea with a respiratory rate of
28/min. The patient is awake but lethargic. His left leg is extremely swollen up to the left groin, red but with some
bluish patches, hot, exquisitely painful and tender. There is no crepitation. In the sole of the foot you note a
suppurating ulcer.

His wife explains to you that her husband was unable to work during the night because he felt very sick. He had
complained about severe pain in his left leg, problems with breathing, and dizziness.

Learning issues

Skin and soft tissue infections


How to recognise the patient with severe infection

Based on the findings from the physical examination and the patient's history, what do you think is wrong with
him, and what immediate actions should be taken?

Learning issues

Sepsis, severe sepsis, and septic shock

Learning issues

PACT module on Sepsis and MODS

After administration of one litre of colloids, blood pressure has returned to a low normal value, but tachycardia
and tachypnoea persist. Antibiotic therapy with amoxicillin/clavulanate had already been started by the doctor in
charge in the Emergency Department after two blood cultures were taken. A deep vein thrombosis of the left leg,
with a lung embolism possibly responsible for the breathing problems and dizziness, was excluded by a Doppler
ultrasound.

Laboratory values show leukocytosis, with 55% band cells and toxic granulation, elevated C-reactive protein,
serum glucose of 22 mmol/l (396.4 mg/dl), creatinine of 215 ÎŒmol/l, a BUN of 31.5 mmol/l, metabolic acidosis
with partial respiratory compensation (pH 7.28, BE –12), and an elevated lactate level of 6 mmol/l. The
coagulation parameters are in the low normal range, and the serum creatinine phosphokinase is normal.

Learning issues

Starting antibiotic treatment

Investigations

Biological markers of infection

How do you interpret these blood results?


What is the likely source of infection?

Learning issues

Involvement of particular sites

NOTE Early clinical differentiation of cellulitis from necrotising fasciitis may be difficult but is crucial.

What are the characteristics of necrotising fasciitis?

Learning issues

Characteristics of necrotising fasciitis

On reading quickly through the patient's history, you find that he was treated several weeks earlier in the
emergency department of a local hospital following an accident with a shard of glass that caused a 3 cm-long cut
in the sole of his left foot. His history also includes non-insulin-dependent diabetes mellitus with micro- and
macroangiopathy, a compensated renal insufficiency, obesity, chronic obstructive pulmonary disease, Severe
infection and cardiomyopathy of unclear origin, serious impairment of the left ventricular ejection fraction, and
atrial fibrillation.

Does the patient's co-morbidity provide diagnostic clues?

Learning issues

Diagnostic approach

Because of the extremely painful pyoderma of the patient's left leg, the history of predisposing factors, and the
association with severe sepsis, you presume the clinical diagnosis of necrotising fasciitis.

What are the mainstays in treatment of necrotising fasciitis?

NOTE Disproportionately severe pain at the site of involvement and clinical signs of septic shock
should raise clinical suspicion of necrotising fasciitis.

NOTE Necrotising fasciitis is a surgical emergency!

The patient is already receiving amoxicillin/clavulanate. Do you judge the antibiotic coverage appropriate?
Please explain your answer.

Learning issues

Optimal antibiotic therapy

Organisms likely to be involved in skin and soft tissue infections

You advise your colleague in the Emergency Department to add clindamycin and a single shot of gentamicin to
the already applied amoxicillin/clavulanate while you try to reach the surgeon on duty.
Unfortunately, the surgeon is busy operating. Emphasising the strongly suspected necrotising fasciitis and the
importance of imperative surgical intervention for source control, you ask him to evaluate the patient as soon as
possible.

What should you do next?

Learning issues

Sepsis guidelines

While waiting for the surgeon on duty, you transfer the patient to the ICU. Again his blood pressure drops to
80/40 mmHg; again you start a fluid challenge with colloids. The patient remains somnolent, tachycardic, and
tachypnoeic with a respiration rate of 30/min. Pulse oximetry shows sufficient arterial oxygenation (94%) with 4 l
O2 by nasal application. The patient has not urinated since arrival at the hospital two hours earlier.

You decide to place a urinary catheter in order to monitor urine production; this produces 350 ml of clear urine.

You cannulate the radial artery for the continuous monitoring of blood pressure and blood controls. The blood
gas analysis shows worsening of the metabolic acidosis, with an elevation of lactate to 8.5 mmol/l.

In addition to a central venous catheter for further CVP-guided fluid resuscitation, you decide to insert a
pulmonary artery catheter to monitor SvO2, CO and wedge pressure, especially in light of the patient's history of
serious impairment of the left ventricular ejection fraction. The initial haemodynamic measurements show a HR
of 135/min and a blood pressure of 90/40 mmHg with a CVP of 13 mmHg, a wedge pressure of 21 mmHg, a CO
of 4.2 l/min and an SvO2 of 58%. The SVR is low.

Learning issues

Metabolic acidosis
PACT module on Homeostasis
PACT module on Haemodynamic monitoring

How do you interpret the further rise in lactate and the haemodynamics?

Learning issues

Metabolic acidosis
PACT module on Homeostasis

You start with dobutamine and note an increase in cardiac output and SvO2, and a rapid fall in the filling
pressures. You continue to administer fluids, CVP- and wedge pressure-guided, to restore adequate circulating
blood volume, and to maintain perfusion pressure. The capillary refill now begins to be prompt.

The surgeon on call arrives to evaluate the patient, but is hesitant to operate because of the patient's unstable
condition. On further discussion, however, he agrees with you that if the patient suffers from necrotising fasciitis,
source control is imperative. You ask for intraoperative Gram stains and cultures, and the surgeon organises the
transport.

Learning issues

Gram stain: advantages, limitations and pitfalls


The surgeon interprets the intra-operative findings as a compartment syndrome due to a marked oedema, and
proceeds with fasciotomy incisions of the left lower extremity.

When the patient returns to the ICU one hour later, he is in poor condition. He is intubated and ventilated, with an
FiO2 of 100%.

The chest radiograph shows bilateral diffuse infiltrates. In addition to dobutamine, norepinephrine (noradrenaline)
is needed in high doses to maintain a sufficient perfusion pressure. The patient is oliguric. His skin now shows
blue streaks reaching to the left groin, and you note some blisters.

Learning issues

PACT module on Respiratory failure


PACT module on Mechanical ventilation
PACT module on Oliguria and anuria

How should you proceed?

Again, you call the surgeon on duty. You stress that the patient is in septic shock and doesn't have signs of
rhabdomyolysis or a history of trauma that would explain a compartment syndrome. Together, you agree to look
for a surgeon experienced in treating necrotising fasciitis.

Learning issues

Septic shock

PACT module on Communication skills

In the meantime, what can you do to improve the patient's respiration and circulation?

Learning issues

PACT module on Respiratory failure


PACT module on Respiratory monitoring

Is there an indication for drotrecogin alfa (recombinant activated protein C)?

Learning issues

Indications and contraindications for the administration of recombinant activated protein C

With a PEEP of 15, and a reduction of the tidal volume to limit the plateau pressure, oxygenation improves
without causing circulatory depression. You order an ACTH stimulation test and start hydrocortisone. You don't
administer recombinant activated protein C because of the planned invasive surgical procedure.
Again, the patient is transferred to the operating room, where this time the entire cutis and subcutis of the left leg,
reaching to the groin, are removed. Fortunately, there is no macroscopic involvement of the deep fascia and
muscles. Cultures are taken of the ulcer on the sole of the foot and of various tissues removed.

What changes would you expect to see in the cutis and subcutis?

What possible surgical problems should you anticipate after the operation, and what measures do you have to
take?

What general critical care complication may ensue?

Learning issues

PACT module on Sepsis and MODS

The second-look operation shows no progression of the necrotising fasciitis. The cultures provide evidence of a
penicillin-sensitive Streptococcus pyogenes, and the antibiotic regimen is adapted to include high-dose penicillin.
Because the ACTH stimulation test does not confirm a relative adrenal insufficiency, hydrocortisone is
discontinued.

In spite of the time lost between the onset of the disease and the beginning of full surgical treatment, as well as
the presence of recognised pre-existing diseases, the patient has an unremarkable acute course. The
vasopressor requirements drop rapidly, the cardiac function stabilises, and only a few courses of renal
replacement therapy are required.

After two weeks, the patient's left leg is covered with skin grafts, and after three weeks the patient is discharged
from the ICU with stable organ functions. He is lucky to have had a relatively short hospital stay and minimal
long-term residual disability.

NOTE The definitive antibiotic regimen should be based on the microbiologic findings.

Learning issues

PACT module on Clinical outcome

On reflection, these two cases presented you with important diagnostic challenges. In the first case, the patient was
transferred to the ICU with severe community-acquired pneumonia caused by Legionella and subsequently developed
complications including ARDS, VAP, acalculous cholecystitis and later meningitis due to sinusitis. This case illustrates
the importance of starting broad-spectrum antibiotic therapy without delay in critically ill patients with severe infection.
The second patient had a history of diabetes mellitus and a small, non-healing wound. It is often tempting to make a
familiar diagnosis for a typical situation. However, it is not always correct.
How does this second case highlight the need for open discussion and exchange of information between
various professionals working in intensive care and acute care scenarios?

S-ar putea să vă placă și